Possible squeeze theorem limit question

  • Thread starter Thread starter physicsjock
  • Start date Start date
  • Tags Tags
    Limit Theorem
Click For Summary
The discussion revolves around applying the squeeze theorem to a sequence defined by inequalities involving terms x_n. The main goal is to demonstrate that the limit of the sequence can be squeezed between two bounds, ultimately converging to a specific value. A point of confusion arises regarding the appearance of the factor 1/3 in the analysis. Participants are exploring how the arithmetic mean (AM) of the previous two terms relates to the limit. Clarifying the derivation of the 1/3 factor is essential for resolving the question.
physicsjock
Messages
84
Reaction score
0
http://img96.imageshack.us/img96/8606/qqqqbj.jpg

so what I'm thinking is that you let

x_n+3 + x_n+2<=x_n+2 + x_n+1 <= x_2 + x_1

then show that the first term can eventually go to the last term

squeezing the term into the middle

but the problem with this is i don't see where the 1/3 comes from

any ideas?
 
Last edited by a moderator:
Physics news on Phys.org
The third term is AM of its previous two terms.
 
Question: A clock's minute hand has length 4 and its hour hand has length 3. What is the distance between the tips at the moment when it is increasing most rapidly?(Putnam Exam Question) Answer: Making assumption that both the hands moves at constant angular velocities, the answer is ## \sqrt{7} .## But don't you think this assumption is somewhat doubtful and wrong?

Similar threads

  • · Replies 5 ·
Replies
5
Views
1K
  • · Replies 5 ·
Replies
5
Views
2K
  • · Replies 12 ·
Replies
12
Views
2K
Replies
2
Views
2K
  • · Replies 25 ·
Replies
25
Views
3K
  • · Replies 3 ·
Replies
3
Views
2K
  • · Replies 2 ·
Replies
2
Views
2K
  • · Replies 21 ·
Replies
21
Views
1K
  • · Replies 24 ·
Replies
24
Views
3K
  • · Replies 2 ·
Replies
2
Views
1K